Biology 111 - Final Questions

Réussis tes devoirs et examens dès maintenant avec Quizwiz!

The membrane protein _____ will catalyze the reaction ADP + P ATP as H+ flow down a gradient from the intermembrane space into the matrix. a. the sodium-potassium pump b. ATP synthase c. cholesterol d. NADH-Q reductase

ATP synthase

Dr. James isolated Staphylococcus aureus, a type of bacteria, from the leg wound of a ten-year-old boy. He suspected these bacteria would grow better at body temperature than room temperature (72°F), but thought that he should collect data to support his thinking. Dr. James introduced the same number of Staphylococcus bacteria into each of six test tubes containing the same type and amount of nutrient broth. Three test tubes were incubated at 98.6°F (Group 1), while three test tubes (Group 2) sat at 72°F. After 24 hours, Dr. James compared the turbidity (indicative of growth) of all six tubes and rated each on a scale of 0 - 4. 0 indicates no turbidity (no growth), while 4 indicates high turbidity (high growth). The following data were collected: Dr. James performed another experiment. Instead of inoculating the test tubes with Staphylococcus, he used the bacterium, Streptococcus. He found that Streptococcus grew better at body temperature than at room temperature. This is a replicate of the first experiment. True False

False

This system of chemicals, H2CO3 -> H+ + IICO3- , act as a buffer in the blood. If hydrogen ions are added to blood which of the following reactions would occur?

H+ + HCO3* -> H2CO3

Study the series of pictures to identify the process as a. facilitated transport. b. active transport. c. passive transport. d. osmosis.

a, facilitated transport.

The haploid (n) number of chromosomes for a human being is a. 23. b. 24. c. 44. d. 46. e. 48.

a. 23.

If the diploid number of chromosomes for an organism is 52, what will the haploid number of chromosomes be? a. 26 b. 48 c. 112 d. 24

a. 26

It is estimated that humans have approximately _____ base pairs in our genome. a. 3 billion b. 2.5 million c. 180 million d. 12 trillion

a. 3 billion

If an element contains eight electrons how many electrons will be placed in the second valence shell? a. 6 b. 2 c. 8 d. 5 e. 11

a. 6

Which of the following substrates is essential to an organism that utilizes photosynthesis? a. 6H2O b. C6H12O6 c. 6O2 d. 6 carbon atoms

a. 6H2O

Which substrate is missing: 6CO2 + _____ + energy = C6H12O6 + 6O2 ? a. 6H2O b. 4H2O c. 4O2 d. glucose

a. 6H2O

One of Chargaff's rules states that a. A = T and G = C. b. A = C and T = G. c. A = G and T = C. d. A = T + G + C.

a. A = T and G = C.

Which statement about the blending concept of inheritance is correct? a. A cross between a tall man and a short woman would always produce children of medium height. b. A cross between a tall man and a short woman can produce some children that are tall while others are short. c. A cross between a tall man and a short woman produces tall children who can then produce short grandchildren. d. Genetic material is unstable which accounts for such a wide diversity in our offspring.

a. A cross between a tall man and a short woman would always produce children of medium height.

Which of the following is true regarding the surface area-to-volume ratio needed for cells to function properly? a. A large surface-area-to-volume ratio is needed to furnish nutrients to and expel wastes from the volume (cytoplasm). b. The surface area should be equal to the volume of cell in order to adequately bring nutrients in and get rid of waste. c. The volume of the cell should be larger than the surface area to ensure that all waste products are effectively removed from the cell. d. The surface-area-to-volume ratio changes constantly. Sometimes the surface area is greater and other times the volume is bigger.

a. A large surface-area-to-volume ratio is needed to furnish nutrients to and expel wastes from the volume (cytoplasm).

What is pleiotropy? a. A single mutant gene affects two or more distinct and seemingly unrelated traits. b. A single mutant gene affects one trait. c. Two mutant genes affect one trait. d. A single mutant gene has no affect on an individual's phenotype.

a. A single mutant gene affects two or more distinct and seemingly unrelated traits.

Which allele combination represents a homozygous dominant individual? a. AA b. Aa c. aa d. None of these are homozygous dominant individuals.

a. AA

The subunits from which ATP is made are a. ADP and phosphate. b. FAD and NAD+. c. FAD and NADPH. d. ADP and FAD. e. ADP and NAD+.

a. ADP and phosphate.

Which of the following domains contains prokaryotes that are known for living in extreme environments? a. Archaea b. Bacteria c. Plantae d. Fungi e. Eukarya

a. Archaea

Which statement is NOT true about bacteria? a. Bacteria contain membrane bound organelles. b. Some are photosynthetic. c. Some are motile due to flagella. d. All bacteria are prokaryotes. e. Bacteria have a cell wall that contains peptidoglycan.

a. Bacteria contain membrane bound organelles.

Photorespiration occurs mainly in a. C3 plants. b. C4 plants. c. CAM plants. d. All of the answer choices.

a. C3 plants.

Which of the following pairs is correct? a. C4 plants = bundle sheath cells contain chlorophyll b. C3 plants = bundle sheath cells contain chlorophyll c. CAM plants = epidermal cells contain chlorophyll d. C4 plants = stoma contain chlorophyll

a. C4 plants = bundle sheath cells contain chlorophyll

Which statement is NOT true about covalent bonds? a. Covalent bonds form when an electron is completely lost or gained from an atom. b. A covalent molecule contains one or more covalent bonds. c. A single covalent bond is drawn as a line between two atoms. d. A pair of electrons is shared between two atoms for each covalent bond. e. Shared electrons allow an atom to complete its valence shell in a covalent molecule.

a. Covalent bonds form when an electron is completely lost or gained from an atom.

For translation to take place, which of the following does NOT need to be present? a. DNA b. mRNA c. tRNA-amino acid complex d. r e. RNA f. ribosome

a. DNA

Choose the correct order of classification from most inclusive to most exclusive. a. Domain-Kingdom-Phylum-Class-Order-Family-Genus-Species b. Kingdom-Domain-Class-Phylum-Order-Family-Genus-Species c. Kingdom-Domain-Class-Phylum-Order-Genus-Species-Family d. Kingdom-Class-Phylum-Domain-Genus-Order-Family-Species

a. Domain-Kingdom-Phylum-Class-Order-Family-Genus-Species

In a classic Mendelian monohybrid cross between a homozygous dominant parent and a homozygous recessive parent, which generation is always completely heterozygous? a. F1 generation b. F2 generation c. F3 generation d. P generation

a. F1 generation

Which of the following coenzymes will accept two electrons and two hydrogen ions during the oxidation of glucose? a. FAD b. NAD+ c. FADH d. NADH

a. FAD

The electrons are unequally shared in _____, and transferred in _____. a. H2O; Na+Cl- b. O2; CH4 c. Na+Cl-; H2O d. CH4; N2

a. H2O; Na+Cl-

Which of the following protein is correctly matched with its location? a. Hemoglobin is found within the red blood cells. b. Myosin is found within the ligaments. c. Actin is found within the tendons. d. Collagen is found within the muscle cells. e. Keratin is found within the cell membrane.

a. Hemoglobin is found within the red blood cells.

Which of the following statements applies to mitosis but not to cytokinesis? a. It contains a checkpoint that will evaluate the quality of the cell. b. It requires a large amount of ATP in order for the cellular mechanisms to function. c. If it is disrupted, it can lead to an abnormal cell. d. It results in two daughter cells that are half the size of the original cell.

a. It contains a checkpoint that will evaluate the quality of the cell.

Determine the number of codon sequences it would require to produce the following amino acid sequence: alanine - proline - serine - isoleucine - stop. a. It would take 5 codon sequences. b. It would take 3 codon sequences. c. It would take 6 codon sequences. d. It would take 4 codon sequences.

a. It would take 5 codon sequences.

Which is NOT true about the genetic code? a. Most amino acids have only one codon. b. It is composed of triplet codes. c. Each codon consists of three bases. d. The groundwork for cracking the code began with the use of synthetic RNA. e. It contains start and stop codons as instructions.

a. Most amino acids have only one codon.

Which type of molecule will not diffuse directly across the cell membrane without the help of transport proteins? a. Na+ b. nonpolar molecules c. carbon dioxide d. oxygen

a. Na+

The fixation of carbon dioxide in the first step of the Calvin cycle is facilitated by which enzyme? a. RuBP carboxylase b. glyceraldehyde-3-phosphate (G3P) c. PEP carboxylase d. lactate dehydrogenase

a. RuBP carboxylase

Which of the following represents a sulfhydryl group? a. SH b. H-N-H c. OH d. 0=C-OH e. C=O

a. SH

What is the best description of technology? a. Technology is the application of scientific knowledge to the interests of humans. b. Technology is the development of new tools. c. Technology is the use of power to make human life easier. d. Technology is the advancement of the functionality of computers.

a. Technology is the application of scientific knowledge to the interests of humans.

Which of the following statements about the Calvin cycle is correct? a. The CO2 is absorbed and reduced to CH2O. b. The CO2 is absorbed and reduced to C2H2O2. c. The CO- is absorbed and reduced to CH2O. d. The CO2 is absorbed and used to form ATP.

a. The CO2 is absorbed and reduced to CH2O.

During the conversion of glucose into a free form of energy only some of the energy is converted into usable ATP. What happens to the rest of the energy? a. The energy is converted to heat. b. The energy is converted to CO2. c. The energy is released in H2O. d. The energy is converted into more glucose.

a. The energy is converted to heat.

Which one is NOT one of the properties of water? Multiple Choice a. The frozen form is more dense than the liquid form. b. Water moderates temperature. c. Water is a solvent. d. Water has a high heat capacity. e. Water has a high heat of evaporation.

a. The frozen form is more dense than the liquid form.

Which of the following situations is most likely to produce a heart attack? a. The gap junctions have collapsed and they do not allow the correct flow of ions from one cell to the next. b. The plasmodesmata have collapsed and they do not allow the correct flow of ions from one cell to the next. c. The tight junction has ripped and is allowing ions to leak through the membrane lining the heart. d. The cell walls have broken down and are not able to support the heart.

a. The gap junctions have collapsed and they do not allow the correct flow of ions from one cell to the next.

Why was the X-ray diffraction photography of Rosalind Franklin and Maurice Wilkins considered critical evidence in the study of DNA? a. The photographs indicated that DNA has a double helix structure. b. The photographs showed equal numbers of purines and pyrimidines. c. The photographs showed the bases of DNA were held together by hydrogen bonds. d. The photographs revealed the structure of the deoxyribose sugar. e. The photographs showed the location of each adenine, guanine, cytosine, and thymine.

a. The photographs indicated that DNA has a double helix structure.

Your professor shows you a computer model of a protein and asks you to identify the highest structural level. It appears to consist of only one polypeptide chain and has a globular shape. What is your determination? a. The protein has tertiary structure. b. The protein has secondary structure. c. The protein has primary structure. d. The protein has quaternary structure. e. This is not a picture of a protein.

a. The protein has tertiary structure.

Which statement is NOT true about eukaryotic chromosomes? a. There is only one chromosome of each type in each body cell. b. Chromosomes contain both DNA and associated histones. c. Chromosomes condense from chromatin at the start of mitosis. d. Chromosomes disperse back into chromatin at the end of mitosis. e. Chromosomes are not located within the nuclear envelope during mitosis.

a. There is only one chromosome of each type in each body cell.

What is the ultimate destination for the energized electrons during the light reactions of photosynthesis? a. They are taken up by NADP+ to produce NADPH. b. They are taken up by NADPH to produce NADP+. c. They are taken up by NADH to produce NAD+. d. They are taken up by NAD+ to produce NADH.

a. They are taken up by NADP+ to produce NADPH.

If a researcher was interested in slowing down the movement of amoebas by disrupting their cell membranes, which protein filaments should she be studying? a. actin b. intermediates c. microtubules d. myosin

a. actin

What type of transport mechanism is required to move sodium ions against their concentration gradient? a. active b. passive c. diffusion d. osmosis

a. active

Cancer cells require many nutrients which are supplied by blood vessels. The growth of new blood vessels into cancerous tissue is called a. angiogenesis. b. metastasis. c. carcinogenesis. d. apoptosis.

a. angiogenesis.

Biotechnology products produced by bacteria include all of the following EXCEPT a. antibodies to deliver radioisotopes to tumor cells. b. clotting factor VIII. c. human growth hormone. d. insulin.

a. antibodies to deliver radioisotopes to tumor cells.

How many atoms are required to form a molecule? a. at least two b. at least three c. at least four d. at least five e. only one

a. at least two

If black fur is produced by a recessive allele, which genotype is most likely to produce a black individual? a. bb b. BB c. Bb d. Cb

a. bb

Which of the following elements would be the most reactive with other elements? a. boron, #5 b. neon, #10 c. argon, #18 d. helium, #2

a. boron, #5

During the light reactions, what structure is responsible for absorbing the solar energy? a. chloroplast b. stroma c. stoma d. bark of the tree

a. chloroplast

What structure is required for an eukaryotic organism to be classified as an autotroph? a. chloroplast b. mitochondria c. nucleus d. Golgi body

a. chloroplast

Homologous structures such as the bones in wings, flippers, and arms would be studied in the field of a. comparative anatomy. b. biogeography. c. the fossil record. d. comparative embryology. e. comparative biochemistry.

a. comparative anatomy.

A dehydration reaction can also be called a(n) _____ reaction since it forms water. a. condensation b. hydrolysis c. isomeric d. energy-releasing e. monomer formation

a. condensation

A transgenic organism a. contains a foreign gene within its genome. b. is produced by cloning a mutant cell. c. acts as the vector for DNA being moved into another organism. d. is produced by the polymerase chain reaction. e. is any genetically modified organism resulting from laboratory research.

a. contains a foreign gene within its genome.

Over the course of millions of years various environments have changed. As grasslands slowly took the place of forests the inhabitants were forced to adapt or they went extinct. During this time period the horse gradually evolved from a small cat sized creature to the size of the modern horses we see today. This is an example of which type of natural selection? a. directional b. disruptive c. stabilizing d. artificial e. drift

a. directional

The event that signals the start of anaphase is a. division of the centromeres to separate sister chromatids. b. migration of the centrioles to opposite poles of the nuclear space. c. a cleavage furrow starts to form. d. asters disappear.

a. division of the centromeres to separate sister chromatids.

From the below table, it is apparent that Bond Energy (kcal/mol) C-C 83 C=C 146 C-O 84 C=O 170 a. double bonds are stronger than single bonds. b. double bonds are weaker than single bonds. c. carbon bonds are the strongest bonds. d. carbon forms only single bonds.

a. double bonds are stronger than single bonds.

Which section of the pre-mRNA contains the protein-coding regions? a. exons c. introns d. The entire pre-mRNA contains protein-coding regions. e. The pre-mRNA does not code for proteins at all.

a. exons

Sugars and amino acids are carried into the cell by means of a. facilitated transport. b. diffusion. c. endocytosis. d. exocytosis.

a. facilitated transport.

What are the end products of photosynthesis? a. glucose and oxygen b. glucose and ATP c. carbon dioxide and energy d. carbon dioxide and water e. oxygen and NADP+

a. glucose and oxygen

Which of the following substrates are required for cellular respiration? a. glucose and oxygen b. glucose and carbon dioxide c. sunlight and oxygen d. sunlight and glucose e. oxygen and carbon dioxide

a. glucose and oxygen

The correct sequence for aerobic metabolic breakdown of glucose is a. glycolysis—preparatory reaction—citric acid cycle—electron transport system. b. preparatory reaction—glycolysis—electron transport—citric acid cycle. c. electron transport system—citric acid cycle—preparatory reaction—glycolysis. d. None of the choices are correct.

a. glycolysis—preparatory reaction—citric acid cycle—electron transport system.

While C4 plants have carbon dioxide fixation and carbon dioxide uptake separated by location within the plant, CAM plants a. have these processes separated by time. b. fix much greater amounts of carbon dioxide than C4 plants. c. are found in tropical regions where nutrients are abundant. d. have these processes separated among two leaf types.

a. have these processes separated by time.

Which functional group when attached to a hydrocarbon chain will form an alcohol? a. hydroxyl b. carbonyl c. carboxyl d. amino e. phosphate

a. hydroxyl

The reason why some individuals who inherit polydactyly (having an extra digit on the hand or feet) do not express the trait is due to a. incomplete penetrance. b. incomplete dominance. c. gene linkage. d. pleiotropy. e. None of the answers are correct.

a. incomplete penetrance.

During _____, the homologous chromosome pairs independently align themselves at the equator of the cell. a. independent assortment b. anaphase I c. crossing-over d. mitosis

a. independent assortment

What type of information does science provide for society? a. information about the natural world b. information about the supernatural world c. information about religious beliefs d. information about religious beliefs and the natural world

a. information about the natural world

Identify "e." a. inner membrane b. matrix c. cristae d. intermembrane space e. outer membrane

a. inner membrane

During the sequencing of DNA into a protein, what is the first step? a. mRNA is copied from a gene. b. A ribosome attaches to the mRNA and signals protein synthesis. c. SRP binds to the signal peptide to shut down synthesis. d. An enzyme removes the signal peptide from the peptide chain.

a. mRNA is copied from a gene.

According to the endosymbiosis theory _____ were engulfed by larger eukaryotic cells. a. mitochondria b. Golgi apparatus c. lysosomes d. ribosomes

a. mitochondria

A cell is to a tissue as an atom is to a a. molecule. b. subatomic particle. c. electron. d. population.

a. molecule.

Fish sperm is composed mostly of DNA. If we tested a sample chemically, we should find relatively high amounts of a. nitrogenous bases, sugar, and phosphate groups. b. phospholipids and steroids. c. amino acids and unsaturated fats. d. triglycerides and ATP. e. globular proteins and stored fats.

a. nitrogenous bases, sugar, and phosphate groups.

Which of the following features is unique to bacterial cells compared to eukaryotic cells? a. nucleoid region b. nucleus c. cell membrane d. cell wall e. ribosome

a. nucleoid region

Which cellular organelle is the most prominent? a. nucleus b. nucleolus c. lysosome d. mitochondria

a. nucleus

The characteristic way in which atoms of an element react is most related to the a. number of electrons in the valence shell. b. number of electrons in the innermost shell. c. number of neutrons in the nucleus. d. size of the nucleus.

a. number of electrons in the valence shell.

A researcher is interested in determining the average length and weight of loblolly pine tree needles in the southeast United States. Will the data be obtained through observation or experimentation? a. observation b. experimentation c. Neither observation nor experimentation. d. Both observation and experimentation.

a. observation

Due to an increased risk of heart disease many doctors have suggested shifting our diet to include more unsaturated fats and less saturated fats. As you cook dinner tonight the recipe calls for two Tbs of oil. Which of the following would be the healthiest choice? a. olive or safflower oil b. pork fat c. melted butter d. lard that was left out at room temperature e. All of these choices are saturated fats.

a. olive or safflower oil

What are the basic structures that make up a nucleotide? a. pentose sugar, phosphate, and nitrogen-containing base b. pentose sugar, nitrate, and phosphorus-containing base c. pentose sugar, phosphate, and sulfur-containing base d. hydroxide group, phosphate, and nitrogen-containing base e. pentose sugar, sodium, and nitrogen-containing base

a. pentose sugar, phosphate, and nitrogen-containing base

Which type of cellular process will most likely be used by an amoeba in order to obtain food? a. phagocytosis b. pinocytosis c. exocytosis d. receptor-mediated endocytosis

a. phagocytosis

Which of the following plants is an example of a CAM plant? a. pineapple b. corn c. rice d. wheat e. oak tree

a. pineapple

During which stage of meiosis does crossing-over occur? a. prophase I of meiosis I b. anaphase I of meiosis II c. telophase I of meiosis I d. prophase II of meiosis II e. anaphase II of meiosis I

a. prophase I of meiosis I

The mass number refers to the number of _____ and _____ within an element. a. protons; neutrons b. protons; electrons c. electrons; neutrons d. protons; molecules e. electrons; atoms

a. protons; neutrons

Acetyl CoA is produced from a. pyruvate and CoA. b. citric acid and CoA. c. ATP and pyruvate. d. CO2 and pyruvate. e. citric acid and CO2.

a. pyruvate and CoA.

Which of the following conditions is necessary to maintain the Hardy-Weinberg equilibrium? a. random mating b. mutations c. sexual selection d. gene flow e. genetic drift

a. random mating

Active transport a. requires an input of ATP. b. is involved in diffusion. c. occurs in osmosis and facilitated transport. d. All of the choices are correct.

a. requires an input of ATP.

If white eyes are produced by a recessive allele, what is the likely genotype of a white-eyed individual? a. rr b. Rr c. RR d. RB

a. rr

Which of the following is a function of a steroid? a. sex hormone b. transmission of genetic information c. long term energy storage d. insulation against cold e. protective layer

a. sex hormone

What is the most plausible explanation for why the male of most bird species have elaborate plumage and mating rituals? a. sexual selection b. genetic drift c. gene flow d. natural selection

a. sexual selection

A polysaccharide is a polymer made up of which kind of monomers? a. simple sugars b. amino acids c. nucleotides d. alternating sugar and phosphate groups e. fatty acids and glycerol

a. simple sugars

Dr. James isolated Staphylococcus aureus, a type of bacteria, from the leg wound of a ten-year-old boy. He suspected these bacteria would grow better at body temperature than room temperature (72°F), but thought that he should collect data to support his thinking. Dr. James introduced the same number of Staphylococcus bacteria into each of six test tubes containing the same type and amount of nutrient broth. Three test tubes were incubated at 98.6°F (Group 1), while three test tubes (Group 2) sat at 72°F. After 24 hours, Dr. James compared the turbidity (indicative of growth) of all six tubes and rated each on a scale of 0 - 4. 0 indicates no turbidity (no growth), while 4 indicates high turbidity (high growth). The following data were collected: The independent (experimental) variable is a. temperature. b. growth of bacteria. c. incubation period. d. amount of initial inoculum.

a. temperature.

Which process produces both NADH and FADH2? a. the citric acid cycle b. glycolysis c. the electron transport system d. fermentation e. the preparatory reaction

a. the citric acid cycle

How do the lipids of the cell membrane and the lipids found in butter and vegetable oil differ? a. the number of fatty acids b. the type of glycerol molecule c. the carbon to carbon bonds d. lipids of the cell membrane are solid at body temperature whereas the lipids found in vegetable oil and butter are liquid at body temperature.

a. the number of fatty acids

At the molecular level, cells from different types of organisms vary in their tolerance to temperature and pH. This variation can mostly likely be attributed to a. the types of enzymes present in the cells. b. ATP efficiency. c. ability of the cells to form glucose polymers. d. replication of nucleic acids. e. extent of saturation of fatty acids.

a. the types of enzymes present in the cells.

Lamarck's inheritance of acquired characteristics included the idea that a. there is a natural force in all living things that pushes them toward perfection. b. local catastrophes cause mass extinctions of species. c. species are only produced through special creation. d. species are fixed and unchanging over time.

a. there is a natural force in all living things that pushes them toward perfection.

All of the following statements are true about restriction enzymes EXCEPT a. they are made by bacteria and viruses. b. hundreds of different ones have been isolated and purified. c. they produce single-stranded complementary ends that can join to two different DNA strands by complementary base-pairing. d. each enzyme cuts DNA at a different specific base sequence.

a. they are made by bacteria and viruses.

What was Darwin's main purpose during his time aboard the HMS Beagle? a. to serve as the ship's naturalist b. to develop evidence that could be used to support the theory of evolution c. to document evidence that would support the concept of natural selection d. Darwin was a sailor with no specific purpose aboard the HMS Beagle.

a. to serve as the ship's naturalist

Which does NOT occur in meiosis? a. two daughter cells at completion b. four daughter cells at completion c. two nuclear divisions d. formation of bivalents

a. two daughter cells at completion

If the atomic number of chlorine is 17 and the atomic mass is 35, how many neutrons are there in the nucleus? a. 17 b. 18 c. 35 d. 70

b. 18

If a parent cell has 48 chromosomes, how many chromosomes will each daughter cell have after mitosis and cytokinesis occurs? a. 24 chromosomes b. 48 chromosomes c. 12 chromosomes d. 96 chromosomes

b. 48 chromosomes

If a woman is a carrier for the color blind recessive allele and her husband is normal, what are their chances that their son will be color blind? a. None since the father is normal. b. 50% since the mother is a carrier. c. 100% because the mother is a carrier. d. None since he will be a carrier like his mother.

b. 50% since the mother is a carrier.

Dr. James isolated Staphylococcus aureus, a type of bacteria, from the leg wound of a ten-year-old boy. He suspected these bacteria would grow better at body temperature than room temperature (72°F), but thought that he should collect data to support his thinking. Dr. James introduced the same number of Staphylococcus bacteria into each of six test tubes containing the same type and amount of nutrient broth. Three test tubes were incubated at 98.6°F (Group 1), while three test tubes (Group 2) sat at 72°F. After 24 hours, Dr. James compared the turbidity (indicative of growth) of all six tubes and rated each on a scale of 0 - 4. 0 indicates no turbidity (no growth), while 4 indicates high turbidity (high growth). The following data were collected: Choose which of the the following statements is a hypothesis for an experiment. a. Based on the data collected during an experiment, S. aureus grows better at body temperature than room temperature. b. Based on observation, it is predicted that S. aureus will grow better at body temperature than at room temperature. c. S. aureus grew equally well at room temperature and at body temperature. d. Based on the data collected during the experiment, it is confirmed that S. aureus grew better at room temperature.

b. Based on observation, it is predicted that S. aureus will grow better at body temperature than at room temperature.

What occurs in anaphase? a. Centrioles move to opposite poles. b. Chromatids are separated and move to opposite poles. c. Chromosomes line up along the equator of the dividing cell. d. The nuclear envelope disappears. e. The nuclear envelope is constructed.

b. Chromatids are separated and move to opposite poles.

During DNA replication, the enzyme _____ catalyzes the elongation of new DNA strands. a. helicase b. DNA polymerase c. DNA ligase d. ATP synthase

b. DNA polymerase

Which best describes the first law of thermodynamics? a. Energy can be created or destroyed and it cannot be changed from one form of energy to another. b. Energy is not created nor destroyed, but it can change from one energy form to another. c. Energy can be created from matter or used to produce matter. d. Some useful energy is lost as heat whenever an energy transfer occurs. e. Energy transfers are always 100% efficient in changing energy from one useful form to another.

b. Energy is not created nor destroyed, but it can change from one energy form to another.

Which stage of the cell cycle focuses on cell growth, replication of organelles, and the accumulation of material for synthesizing DNA? a. M phase b. G1 phase c. G2 phase d. S phase e. G0 phase

b. G1 phase

Which is NOT true about the electrical charges of elements? a. Protons carry a positive charge. b. In an atom, the number of protons and neutrons must be equal. c. An atom is neutral when the positive and negative charges balance. d. Neutrons have no electrical charge.

b. In an atom, the number of protons and neutrons must be equal.

Why is crossing-over important? a. It provides extra genetic material for the daughter cells. b. It increases the likelihood that daughter cells contain different genetic material. c. It produces the proteins that are associated with DNA in chromosomes. d. It increases chromosome condensation. e. It separates the homologous chromosomes.

b. It increases the likelihood that daughter cells contain different genetic material.

Cutting off the tails of mice over several generations should cause the tails of their offspring to become shorter, according to the ideas of which of these scientists? a. Darwin b. Lamarck c. Cuvier d. Lyell e. Wallace

b. Lamarck

Which of the following concepts is NOT one of the basic theories of biology? a. All organisms are composed of cells. b. Life may arise through spontaneous generation. c. New cells only come from preexisting cells. d. The internal environment of an organism stays relatively constant. e. All living things have a common ancestor and are adapted to a particular way of life.

b. Life may arise through spontaneous generation.

In his theory of natural selection, Darwin incorporated the premise that available resources were not sufficient for all members of a population to survive. Darwin adopted these ideas of competition and survival from a. Erasmus Darwin. b. Malthus. c. Cuvier. d. Lamarck. e. Plato.

b. Malthus.

Cell biologists have introduced radioactively labeled carbohydrates, fats, and amino acids to living cells, allowed time for cell metabolism, and then placed the cells in a blender and separated out the various cell organelles from the cytoplasm. They found the radioactive molecules could soon be detected as part of various cellular compounds, although the visible cell structures appeared unchanged. Which of the following is the best conclusion based on this information? a. These "food" molecules are used only for energy. b. Most cell components are constantly being broken down and rebuilt. c. Radioactivity was transferred from the introduced molecules to the resident stable molecules. d. Molecules diffuse at random through the cytoplasm and into cell organelles. e. Radioactivity has an unusual role in the metabolism of living cells.

b. Most cell components are constantly being broken down and rebuilt.

Which of the following statements is NOT true according to Mendel's law of segregation? a. Each individual contains two factors for each trait. b. One factor must be dominant and one factor recessive in each individual. c. Factors separate from each other during gamete formation. d. Each gamete contains one copy of each factor. e. Fertilization restores the presence of two factors.

b. One factor must be dominant and one factor recessive in each individual.

Which statement is true about RNA? a. It contains adenine paired to thymine. b. One of the bases from DNA is replaced by uracil. c. It contains the sugar deoxyribose. d. Its nucleotides contain twice as many phosphate groups as DNA's nucleotides. e. It is a double-stranded molecule.

b. One of the bases from DNA is replaced by uracil.

Which of the following is true about the differences between organic and inorganic molecules? a. Organic molecules are living and inorganic molecules are nonliving. b. Organic molecules contain at least carbon and hydrogen and inorganic molecules do not. c. Organic molecules are relatively unstable and inorganic molecules are relatively stable. d. Organic molecules are held together primarily with ionic bonds and inorganic molecules are held together primarily with covalent bonds.

b. Organic molecules contain at least carbon and hydrogen and inorganic molecules do not.

You come across an article that discusses a new drug that interferes with hydrogen bonding during protein formation. Based on your understanding of the levels of protein organization, which of the following is a probable consequence following exposure to this new drug? a. The drug would prevent amino acids from bonding together which would in turn prevent the formation of the primary structure of the protein. b. Protein folding will arrest during the formation of secondary structure. The drug would prevent the polypeptide from forming an alpha helix or a beta sheet. c. Protein folding will arrest during the formation of tertiary structure. The drug would interfere with the ability of the alpha helix or beta sheet to form the bonds necessary to fold into a globular protein. d. The polypeptide would be unable to form a quarternary structure because it would be unable to form the bonds necessary to link up with other protein molecules.

b. Protein folding will arrest during the formation of secondary structure. The drug would prevent the polypeptide from forming an alpha helix or a beta sheet.

Which of the following structures is classified as a nucleic acid? a. myosin b. RNA c. glucose d. glycerol e. cytosine

b. RNA

Which of the following statements about DNA replication is NOT correct? a. Unwinding of the DNA molecule occurs as hydrogen bonds break. b. Replication occurs as each base is paired with another exactly like it. c. The process is known as semiconservative replication because one old strand is conserved in the new molecule. d. The enzyme that catalyzes DNA replication is DNA polymerase. e. Complementary base pairs are held together with hydrogen bonds.

b. Replication occurs as each base is paired with another exactly like it.

If one strand of DNA has the base sequence AAG-CAA, the complementary strand has which of the following sequences? a. UUC-GUU b. TTC-GTT c. AAG-CAA d. UTC-GTU e. TTC-GTG

b. TTC-GTT

Which is the best definition of osmosis? a. The movement of molecules from an area of their higher concentration to an area of their lower concentration. b. The movement of water across a semipermeable membrane from an area of higher water concentration to an area of lower water concentration. c. The movement of molecules from an area of their lower concentration to an area of their higher concentration. d. The movement of water across a semipermeable membrane from an area of lower water concentration to an area of higher water concentration. e. The movement of a substance against its concentration gradient through the release of energy from ATP.

b. The movement of water across a semipermeable membrane from an area of higher water concentration to an area of lower water concentration.

DNA is housed within the nucleus, but the mRNA code needs to leave in order to be translated into a protein. How does the mRNA code leave the nucleus if the nucleus is surrounded by the nuclear envelope? a. The nuclear envelope dissolves in order to allow the mRNA code to leave. b. The nuclear envelope contains pores which provide passage for the mRNA. c. A vesicle will bud off from the nuclear envelope containing the mRNA. d. The mRNA will be coded into a protein within the nucleus. Then the protein leaves the nucleus.

b. The nuclear envelope contains pores which provide passage for the mRNA.

Characterize these reactions involved in the Calvin cycle. a. These are reactions involved in the regeneration of RuBP. The reactions include an exergonic reaction (ATP breaks down into ADP) and the reduction of the coenzyme NADPH. b. These reactions are involved in the reduction of CO2. Energy is required in the form of ATP, which is hydrolyzed to ADP. The substrate BPG is reduced, while the coenzyme NADPH is oxidized. c. These reactions are involved in the fixation of CO2. Energy is required in the form of ADP, which is hydrolyzed to ATP. The substrate BPG is oxidized, while the coenzyme NADPH is reduced. d. These reactions are part of the light reactions in a noncyclic pathway. ATP and NADPH are used during these reactions.

b. These reactions are involved in the reduction of CO2. Energy is required in the form of ATP, which is hydrolyzed to ADP. The substrate BPG is reduced, while the coenzyme NADPH is oxidized.

Which of the following is not a characteristic of homologous chromosomes? a. They are similar in size. b. They carry the same alleles for all traits. c. They carry genes for the same traits. d. They are similar in shape and location of the centromere.

b. They carry the same alleles for all traits.

Which of the following is NOT true about cancer cells? a. They never fully differentiate. b. They exhibit contact inhibition. c. They exhibit uncontrolled growth. d. They exhibit disorganized growth. e. They may undergo metastasis.

b. They exhibit contact inhibition.

Natural selection was independently proposed as a means of evolution by Darwin and a. Lyell. b. Wallace. c. LeClerc. d. Lamarck.

b. Wallace.

In the Watson and Crick model of DNA, the "steps" of the ladder are composed of a. sugars. b. a purine and a pyrimidine. c. two purines. d. two pyrimidines. e. a sugar and a phosphate molecule.

b. a purine and a pyrimidine.

What are alleles? a. genes for different traits, such as hair color or eye color b. alternative forms of a gene for a single trait, such as blue eyes or brown eyes c. the locations of genes on a chromosome d. the recessive form of a gene e. the dominant form of a gene

b. alternative forms of a gene for a single trait, such as blue eyes or brown eyes

You come across an image of a molecule in research article about fatty acids. You notice the molecule has several double bonds in its structure. Based on your knowledge of fatty acids, you conclude that this molecule is most likely a. a saturated fatty acid. b. an unsaturated fatty acid. c. something other than a fatty acid because fatty acids don't have double bonds.

b. an unsaturated fatty acid.

During which stage of meiosis do the homologous chromosomes separate? a. prophase I b. anaphase I c. telophase I d. prophase II e. anaphase II

b. anaphase I

A(n) _____ is a group of three bases on tRNA that is complementary to a specific mRNA codon. a. codon b. anticodon c. poly-A tail d. cap

b. anticodon

Transgenic _____ have been given suicide genes that cause them to self-destruct when the job for which they are engineered has been accomplished. a. animals b. bacteria c. plants d. fungi e. All of the choices are correct.

b. bacteria

Plants show turgor pressure when a. cells are losing water from their water vacuoles. b. cells contain water vacuoles that are full of water. c. water is being used up in photosynthesis. d. water is being evaporated from the leaves.

b. cells contain water vacuoles that are full of water.

Which of the following carbohydrates would NOT be a molecule used for energy storage? a. starch b. cellulose c. glycogen d. All of the above are used for energy storage

b. cellulose

Which carbohydrate is found in the exoskeleton of insects and crabs? a. starch b. chitin c. cellulose d. glycogen e. glycerol

b. chitin

Which term refers to the attraction of water molecules to one another? a. adhesion b. cohesion c. hydrolysis d. photolysis e. polarity

b. cohesion

Which of the following would NOT contribute to genetic variation? a. crossing-over of homologous chromosomes b. crossing-over of sister chromatids c. the random alignment of the chromosomes during metaphase I d. the combination of sperm and egg genes

b. crossing-over of sister chromatids

At the beginning of a research project on egg laying in seagulls, a researcher noted that the birds laid an average of 7-9 eggs per clutch. After studying the population for 15 generations, the researcher noted that the birds were now laying 3-4 eggs per clutch. What type of natural selection is occurring in the population of gulls? a. stabilizing b. directional c. disruptive d. There is no selection occurring in this population.

b. directional

The following picture depicts which of the following changes in chromosome structure? a. deletion b. duplication c. translocation d. aneuploidy

b. duplication

Which process is responsible for moving cellular wastes across the cell membrane and out of the cell? a. endocytosis b. exocytosis c. pinocytosis d. receptor-mediated endocytosis

b. exocytosis

If an animal needed to store energy for long-term use which of the following would be the best choice? a. fructose and glucose in the form of honey b. fat molecules c. complex cellulose molecules d. starch e. glycogen with extensive side branches of glucose

b. fat molecules

How many molecules of water are used during hydrolysis to break the following polypeptide into its constituent amino acids: alanine-leucine-tryptophan-glycine-valine-alanine? a. six b. five c. one d. seven

b. five

Which of these processes occurs in the cytosol? a. the citric acid cycle b. glycolysis c. the electron transport system d. the preparatory reaction

b. glycolysis

The first phase of cellular respiration is a. the citric acid cycle. b. glycolysis. c. the electron transport system. d. fermentation. e. the preparatory reaction.

b. glycolysis.

Without a cytoskeleton, eukaryotic cells would NOT a. synthesize protein. b. have an efficient way to transport materials from one organelle to another. c. have an efficient means of metabolism. d. communicate with adjacent cells.

b. have an efficient way to transport materials from one organelle to another.

Which genotype is most likely to survive in a region of the world that contains malaria? a. homozygous recessive for hemoglobin shape b. heterozygous for hemoglobin shape c. homozygous dominant for hemoglobin shape d. All genotypes have the same potential for survival in a region that contains malaria.

b. heterozygous for hemoglobin shape

What is the first step in producing a transgenic animal? a. implantation of a donor egg into the host mother b. inject the desired gene(s) into a donor egg c. transgenic organism produces offspring that have the desired gene which can then produce more transgenic offspring d. transgenic eggs are harvested from the surrogate mother

b. inject the desired gene(s) into a donor egg

The function of a vector in genetic engineering is to a. cut DNA into many fragments. b. introduce rDNA into a host cell. c. link together newly joined fragments of DNA. d. make millions of copies of a specific segment of DNA. e. separate fragments of DNA by their length and electrical charges.

b. introduce rDNA into a host cell.

What term is used for molecules that have identical molecular formulas but the atoms in each molecule are arranged differently? a. isotope b. isomer c. homomolecules d. organic e. balanced

b. isomer

DNA replication is considered semiconservative because a. it will create three new identical strands when finished. b. it uses one old strand as a template for the synthesis of a new strand. c. it always replicates in the 3' to 5' prime direction. d. it never replicates in the 5' to 3' prime direction.

b. it uses one old strand as a template for the synthesis of a new strand.

Each time the water in a cell freezes slowly, long sharp crystals spear through the membrane structures of the cell. However, in the frozen state, virtually no chemical reactions occur. The most likely explanation for the bad taste of meat that has "freezer burn" from repeated freezing is the destruction of a. the Golgi bodies and their vesicles. b. lysosomes, which results in the cell digesting itself. c. rough endoplasmic reticulum causing the release of ribosomes. d. ribosomes causing them to break into subunits. e. the nuclear membrane causing mixing of nucleoplasm and cytoplasm.

b. lysosomes, which results in the cell digesting itself.

Identify "b." a. inner membrane b. matrix c. cristae d. intermembrane space e. outer membrane

b. matrix

During cellular movement which of the following filaments will be the ones that are responsible for attaching and pulling the other filaments along? a. actin b. myosin c. trypsin d. intermediate filaments

b. myosin

The region that contains the genetic information in a bacterial cell is called the a. nucleus. b. nucleoid. c. nucleolus. d. nucleosome. e. nucleoprotein.

b. nucleoid.

What structures enable the vesicles from the Golgi apparatus to fuse with the cell membrane? a. membrane-associated proteins b. phospholipids c. cholesterol molecules within the cell membrane d. secretory vesicles that engulf the Golgi apparatus will allow the vesicles to fuse

b. phospholipids

Chloroplasts are to _____ as _____ are to aerobic respiration. a. stroma; cristae b. photosynthesis; mitochondria c. thylakoid membranes; matrix d. protein synthesis; lysosomes

b. photosynthesis; mitochondria

In lab, your biology instructor has asked you to identify an unknown solution. During one of the tests you perform your sample turns light purple indicating the presence of peptide bonds. Based on this information, you determine your solution contains a. lipids. b. protein. c. starch. d. DNA. e. monosaccharides.

b. protein.

Most bacteria live in a solution that is hypotonic relative to the cytoplasm of the bacteria. This means that the bacterial cell is constantly taking in water. The cell wall of a bacterium is a peptidoglycan polymer that is tightly cross-linked. This would therefore function to a. regulate the flow of most molecules into and out of the bacterial cell. b. provide a rigid wall that prevents the cell from swelling. c. provide a rigid wall that prevents the cell from shrinking. d. confirm a close relationship to plant cells that have a similar structure and live in hypotonic solutions. e. make all bacteria fairly uniform in metabolic chemistry.

b. provide a rigid wall that prevents the cell from swelling.

The most common source of genetic variation in sexually reproducing organisms is a. mutation. b. recombination of alleles through meiosis and fertilization. c. duplication of chromosomes. d. duplication of genes.

b. recombination of alleles through meiosis and fertilization.

What cellular organelle is responsible for synthesizing proteins based on the information in a mRNA molecule? a. Golgi body b. ribosome c. nucleus d. lysosome

b. ribosome

The alpha helix and beta sheet are found at which level of protein organization? a. primary structure b. secondary structure c. tertiary structure d. quaternary structure

b. secondary structure

Required information Use figures A, B, and C to answer the following questions. Figure A is an 8-cm. cube. Figure B is eight, 4-cm cubes. Figure C is sixty-four, 2-cm cubes. As a cell increases in size, the a. surface-area-to-volume ratio increases. b. surface-area-to-volume ratio decreases. c. surface-area-to-volume stays the same. d. change in surface area relative to volume is unpredictable and depends on the type of cell.

b. surface-area-to-volume ratio decreases.

A doctor is testing the effectiveness of a new antibiotic. He gives the first group of patients a placebo, a second group receives antibiotic A, while the third group receives antibiotic B. Which of the groups is considered the control group? a. the group that received antibiotic A b. the group that received the placebo c. the group that received antibiotic B d. both groups receiving antibiotic A and B

b. the group that received the placebo

A valence shell is best described as a. the electron shell closest to the nucleus. b. the outermost electron shell of an atom. c. the volume of space in which electrons are most often found. d. the original energy level of electrons in photosynthesis.

b. the outermost electron shell of an atom.

The purpose of therapeutic cloning is a. to produce an individual to the donor of the nucleus. b. to produce specialized tissue cells. c. to stimulate cells that have been arrested in the S phase. d. None of the answer choices are correct.

b. to produce specialized tissue cells.

In what kind of classic Mendelian cross would you expect to find a ratio of 9:3:3:1 among the F2 offspring? a. monohybrid cross b. dihybrid cross c. testcross d. None of the choices are correct.

b.dihybrid cross

A solution with a pH of 7.0 has _____ times _____ H+ than a solution of pH 10. a. 30; more b. 300; less c. 1000; more d. 1000; less e. None of these are correct.

c. 1000; more

Which allele combination represents a heterozygous individual? a. aa b. AA c. Aa d. None of these represent a heterozygous individual.

c. Aa

Which statement is NOT true about ionic bonds? a. One atom acts as an electron donor and another atom acts as an electron acceptor. b. Electrons are completely lost or gained in ion formation. c. An ion has the same number of electrons as a nonionic atom of the same element. d. An ionic bond occurs between positive ions and negative ions. e. A salt such as NaCl is formed by an ionic reaction.

c. An ion has the same number of electrons as a nonionic atom of the same element.

What is the molecular formula for five glucose (C6H12O6) molecules bonded together by dehydration synthesis? a. C30H50O26 b. C30H60O30 c. C30H52O26 d. C30H50O30

c. C30H52O26

Which cytoskeletal element is NOT correctly associated with its characteristic? a. Cilia are small extensions of membrane-surrounded microtubules. b. Microtubules are made up of a globular protein called tubulin. c. Centrioles are found in the microtubule organizing centers of plants. d. Flagella have a nine + two pattern of microtubule structure. e. Basal bodies are located at the base of cilia and flagella.

c. Centrioles are found in the microtubule organizing centers of plants.

An unknown chemical is analyzed and found to contain the bases thymine and guanine. This chemical is most likely a. tRNA. b. mRNA. c. DNA. d. rRNA.

c. DNA.

DNA was shown to be the transforming substance when only the _____ enzymes could inhibit transformation. a. proteinase b. RNAase c. DNAase d. lipase

c. DNAase

All of the following are true about the chromosomes of a multicellular organism EXCEPT: a. They are made up of DNA and protein. b. Each chromosome is replicated into two chromatids during the S phase of interphase. c. Each chromosome separates into two daughter chromosomes by binary fission. d. All cells contain chromosomes that carry the same genetic information.

c. Each chromosome separates into two daughter chromosomes by binary fission.

Which of the following is not part of Mendel's law of segregation? a. Each individual has two factors for each trait. b. The factors segregate during meiosis. c. Each gamete will carry two factors for a trait. d. Fertilization gives the offspring two factors for each trait.

c. Each gamete will carry two factors for a trait.

Required information Use figures A, B, and C to answer the following questions. Figure A is an 8-cm. cube. Figure B is eight, 4-cm cubes. Figure C is sixty-four, 2-cm cubes. What figure has the greatest surface area (height x width x number of sides x number of cubes)? a. Figure A b. Figure B c. Figure C d. All of these are the same.

c. Figure C

Required information Use figures A, B, and C to answer the following questions. Figure A is an 8-cm. cube. Figure B is eight, 4-cm cubes. Figure C is sixty-four, 2-cm cubes. Which figure has the greatest surface-area-to-volume ratio? a. Figure A b. Figure B c. Figure C d. All of these are the same.

c. Figure C

Insulin is synthesized in significant quantities only in beta cells in the pancreas. When the beta cell is appropriately stimulated by rising blood glucose levels, insulin is secreted from the cell by exocytosis. What organelle would be responsible for the packaging and transporting the insulin out of the cell? a. nucleus b. ribosomes c. Golgi apparatus d. lysosomes e. centrioles

c. Golgi apparatus

Which of the following gives rise to both lysosomes and vesicles? a. rough endoplasmic reticulum b. mitochondria c. Golgi apparatus d. ribosomes e. centrioles

c. Golgi apparatus

In the cell pictured, there is no net movement of water. The amount leaving the cell and entering the cell is the same. In what type of environment is this cell found? a. hypertonic b. hypotonic c. isotonic d. None of the above.

c. Isotonic

Some biologists study the complex interactions of animals and plants in forests or prairies. Such ecology field research often produces slightly different results for different researchers. In contrast, ecology experiments that are run indoors with one organism in a terrarium usually produce results that are repeatable. What is the most likely explanation? a. The scientific method is only useful in laboratory settings. b. It is not possible to establish a control group outside of a laboratory. c. It is easier to hold all but one variable constant in a laboratory. d. Field research is only descriptive, and descriptive research is not strictly "science." e. Fieldwork is inductive; lab work is deductive.

c. It is easier to hold all but one variable constant in a laboratory.

Which of the following is NOT offered as evidence in support of the endosymbiotic theory, which proposes that eukaryotic cells have evolved as a "committee" of prokaryotic cells? a. Mitochondria and chloroplasts are similar in size and structure to some species of bacteria. b. The ribosomes of chloroplasts and mitochondria are similar to bacteria. c. Mitochondria and chloroplasts can actively break away from eukaryotic cells and live on their own. d. Mitochondria and chloroplasts have their own DNA coding separate from the eukaryotic nucleus. e. All of the choices offer support of the endosymbiotic theory.

c. Mitochondria and chloroplasts can actively break away from eukaryotic cells and live on their own.

Which is NOT a characteristic of mitochondria? a. The folded membrane in mitochondria forms cristae. b. Mitochondria are the site of cellular respiration. c. Mitochondria have a single membrane for cellular respiration. d. Mitochondria contain DNA and ribosomes. e. The inner space of the mitochondrion contains a fluid matrix.

c. Mitochondria have a single membrane for cellular respiration.

Which of the following is/are true about natural selection? a. It acts on genotypes rather than phenotypes. b. It assures the survival of every fit individual. c. On average, it favors the survival of individuals that have adaptive characteristics. d. It always selects for more complex forms. e. All of the choices are correct.

c. On average, it favors the survival of individuals that have adaptive characteristics.

Transcription begins when a. initiation factors assemble ribosomal subunits, mRNA, and initiator tRNA. b. RNA polymerase comes to a stop sequence. c. RNA polymerase binds to a region of DNA called the promoter. d. new nucleotides are added to an existing strand of nucleotides.

c. RNA polymerase binds to a region of DNA called the promoter.

Most of Darwin's observations about changes in species over time and in different environments took place in and near a. North America. b. Africa. c. South America. d. Asia. e. Greenland.

c. South America.

Which of the following is a true statement concerning meiosis in males and females? a. Males provide more genes in sperm than females provide in eggs. b. Crossing-over occurs more often in the formation of sperm than in eggs. c. Spermatogenesis in males results in four functional sperm while oogenesis in females results in only one egg. d. Sperm that contain a recombination of genes are usually more successful in fertilizing an egg.

c. Spermatogenesis in males results in four functional sperm while oogenesis in females results in only one egg.

Required information Use figures A, B, and C to answer the following questions. Figure A is an 8-cm. cube. Figure B is eight, 4-cm cubes. Figure C is sixty-four, 2-cm cubes. If the figures represented cells, what size cell has the greatest potential to bring nutrients in and rid itself of waste? a. The individual cells in Figure A. b. The individual cells in Figure B. c. The individual cells in Figure C. d. All cells have the same potential.

c. The individual cells in Figure C.

What is the function of polar bodies? a. They nurse the egg as it leaves the follicle. b. They orient the sperm toward the egg. c. They allow a reduction in chromosomes while preserving most of the cytoplasm for one egg. d. They orient the egg for penetration by the sperm.

c. They allow a reduction in chromosomes while preserving most of the cytoplasm for one egg.

What is the function of naturally occurring restriction enzymes in bacterial cells? a. They separate the DNA strands during DNA replication. b. They cut the bacterial cell's DNA. c. They cut any viral DNA that enters the cell. d. They reattach pieces of DNA into a continuous molecule after DNA replication.

c. They cut any viral DNA that enters the cell.

Transcription of a DNA molecule with a nucleotide sequence of AAA-CAA-CTT results in an mRNA molecule with the complementary sequence of a. GGG-AGA-ACC. b. UUU-CUU-CAA. c. UUU-GUU-GAA. d. TTT-GAA-GCC. e. CCC-ACC-TCC.

c. UUU-GUU-GAA.

ATP is considered to be a. an enzyme used widely in all kinds of cells. b. a coenzyme used to inhibit or activate different enzymes. c. a molecule that carries a great deal of chemical energy in a chemical bond. d. the precursor of a high-energy membrane-bounded protein.

c. a molecule that carries a great deal of chemical energy in a chemical bond.

The polar body is a. another name for an egg cell. b. a precursor cell that becomes an egg cell. c. a nonfunctional cell formed at the same time as an egg cell. d. the cell produced when fertilization occurs.

c. a nonfunctional cell formed at the same time as an egg cell.

A coenzyme is a. an inorganic ion that interacts with an enzyme to allow it to work. b. a protein-based organic molecule that interacts with an enzyme to allow it to work. c. a nonprotein organic molecule that interacts with an enzyme to allow it to work. d. an inorganic ion that interacts with an enzyme to inhibit it.

c. a nonprotein organic molecule that interacts with an enzyme to allow it to work.

The expected outcome of an experiment is known as a. a scientific model. b. an experiment. c. a prediction. d. a scientific theory or principle. e. experimental results.

c. a prediction.

To what does the term grana refer? a. the double membrane that surrounds the chloroplast b. a flattened disk or sac in the chloroplast c. a stack of thylakoid membranes d. the fluid-filled space of the chloroplast e. the pores on the underside of a leaf that facilitate gas exchange

c. a stack of thylakoid membranes

Over many generations man has chosen certain desirable characteristics in dogs by allowing dogs possessing those characteristics to reproduce. In this way, 150 breeds of dogs, all the same species, have descended from wolves. This process is referred to as a. natural selection. b. inheritance of acquired traits. c. artificial selection. d. convergent evolution.

c. artificial selection.

Endergonic reactions a. release energy. b. have a negative ΔG and occur spontaneously. c. can only occur if there is an input of energy. d. have products with less free energy than the reactants. e. All of the choices are correct.

c. can only occur if there is an input of energy.

Organic molecules are those that contain at least a. carbon. b. carbon and oxygen. c. carbon and hydrogen. d. carbon, oxygen, and hydrogen.

c. carbon and hydrogen.

Which is most closely associated with the Calvin cycle? a. ATP production b. oxygen production c. carbon dioxide fixation d. carbon dioxide production e. removal of electrons from water for passage through an electron transport system

c. carbon dioxide fixation

Which carbohydrate is found in the cell walls of plants? a. starch b. chitin c. cellulose d. glycogen e. glycerol

c. cellulose

Which of the following protein functions is not correctly associated with the correct integral protein? a. carrier proteins—facilitate passage of molecules through the membrane b. enzymatic proteins—catalyze a specific reaction c. channel proteins—block the activity of carrier proteins d. cell recognition proteins—recognize pathogens

c. channel proteins—block the activity of carrier proteins

A multicellular organism, such as a rabbit, is a. composed of many cells that are all the same type. b. composed of a single cell. c. composed of a variety of different cell types. d. composed of one giant cell.

c. composed of a variety of different cell types.

Identify "a." a. grana b. matrix c. cristae d. intermembrane space e. outer membrane

c. cristae

A certain species of butterfly varies in color from white to dark blue. The birds found in the same area feed on the white or lightly colored butterflies, leaving butterflies that are darkly colored. This is an example of what type of selection? a. stabilizing selection b. disruptive selection c. directional selection d. artificial selection

c. directional selection

A university biology department wishes to hire a scientist to work on the relationships among the wolves, moose, trees, and physical features on an island. If you were charged with writing the job description, you should title the position a. population geneticist. b. molecular biologist. c. ecosystem ecologist. d. organismal physiologist. e. island zoologist.

c. ecosystem ecologist.

Study the figure shown here. What does the letter "A" depict? a. energy of the reactant b. energy of the product c. energy of activation d. substrate concentration

c. energy of activation

A hydrocarbon is hydrophobic a. at all times. b. only in the living cell environment. c. except when it has a polar functional group attached. d. in carbohydrates but not in lipids.

c. except when it has a polar functional group attached.

The function of sunlight in photosynthesis is to a. reduce NADPH to NADP. b. fix O2 in the Calvin cycle. c. excite electrons in chlorophyll. d. combine carbon dioxide and water to form ATP and NADPH. e. split water releasing oxygen.

c. excite electrons in chlorophyll.

Which of the following features were present in the transitional species between amphibian tetrapods and the fishes? a. fins, rounded head with eyes on the side, and a neck lungs, fins, and scales b. a flat head with eyes on the top, fins, and scales c. fins, simple wrist bones, and expanded ribs

c. fins, simple wrist bones, and expanded ribs

Darwin's assumption that living forms must be descended from extinct forms was based on ________ evidence. a. anatomical b. biochemical c. fossil d. biogeographical

c. fossil

A random alteration in the sequence of DNA nucleotides that provides a new variant allele is known as a a. gene mutation. b. polymorphism. c. gene frequency. d. disruption.

c. gene mutation.

The introduction of normal genes into an afflicted individual for therapeutic use is called a. human cloning. b. proteomics. c. gene therapy. d. genetic profiling.

c. gene therapy.

A product of photosynthesis, _____, is the chief source of energy for most organisms. a. oxygen b. sucrose c. glucose d. Correct e. All of the answer choices.

c. glucose

_____ is a product of the Calvin cycle that is used to form glucose phosphate, amino acids or fatty acids. a. ATP b. ribulose bisphosphate (RuBP) c. glyceraldehyde-3-phosphate (G3P) d. PEP carboxylase (PEPcase) e. carbon dioxide

c. glyceraldehyde-3-phosphate (G3P)

A coastal climate is moderated primarily by which of the following properties of water? Water a. is the universal solvent. b. is cohesive and adhesive. c. has a high heat of evaporation. d. has a high surface tension.

c. has a high heat of evaporation.

Upon examination, a person is found to only have half the normal number of LDL-cholesterol receptors. Which of the following best explains this? a. pleiotropic inheritance b. polygenic inheritance c. incomplete dominance d. X-linked inheritance

c. incomplete dominance

All of the following are examples of damage caused by acid deposition from rain EXCEPT a. leaching of aluminum from the soil into lakes which results in the formation of toxic methyl mercury from mercury in the lake sediments. b. weakens trees in the forests and kills seedlings. c. increased agricultural yields. d. damage to marble and limestone monuments.

c. increased agricultural yields.

For five years, you wake up before the alarm is set to ring each morning. This leads you to conclude that all people have a built-in "alarm clock" capable of waking them up. From a science viewpoint, this conclusion a. is science because it is based on real observations. b. is science because it is predictive of what will happen tomorrow morning. is scientifically valid because 5 years × 365 days is a large number of trials. c. may not be valid because it generalizes about all people, and there may have been other variables that could awaken you without a built-in clock. d. cannot be scientifically tested because it involves human behavior.

c. may not be valid because it generalizes about all people, and there may have been other variables that could awaken you without a built-in clock.

What phase of mitosis is pictured? a. prophase b. prometaphase c. metaphase d. anaphase e. telophase

c. metaphase

Which of the following conditions is not necessary to maintain the Hardy-Weinberg equilibrium? a. large gene pool b. no migration c. mutation d. random mating e. no selection

c. mutation

Adult humans cannot synthesize _____ out of the _____ common amino acids. a. eleven; twenty b. nine; eleven c. nine; twenty d. any; twenty e. half; all

c. nine; twenty

What are the end products of photosynthesis? a. water and carbon dioxide b. water and oxygen c. oxygen and carbohydrate d. carbohydrate and water

c. oxygen and carbohydrate

A common method used to introduce rDNA into bacterial host cells is a. viral infection. b. bacteriophage infection. c. plasmid transfer. d. microinjection of DNA. e. laser irradiation of cells in a fluid containing DNA.

c. plasmid transfer.

All the members of a single species that occupy a particular area at the same time are known as a a. subspecies. b. gene pool. c. population. d. group. e. sub-population.

c. population.

Which sequence of the mitotic stages is correct? a. prophase, anaphase, prometaphase, metaphase, telophase b. prophase, telophase, anaphase, prometaphase, metaphase c. prophase, prometaphase, metaphase, anaphase, telophase d. telophase, anaphase, prophase, prometaphase, metaphase anaphase, prometaphase, metaphase, prophase, telophase

c. prophase, prometaphase, metaphase, anaphase, telophase

The function of DNA ligase in recombinant technology is to a. cut DNA into many fragments. b. carry DNA into a new cell. c. seal DNA into an opening created by restriction enzymes. d. make millions of copies of a specific segment of DNA. e. separate fragments of DNA by their length and electrical charges.

c. seal DNA into an opening created by restriction enzymes.

All of the following are associated with the mitochondria except: a. ATP production b. cristae c. stroma d. matrix

c. stroma

Cytokinesis in plant cells differs from animal cells because a. the lysosomes within the plant cell forms a cell plate. b. microtubules are used to form the cell plate. c. the Golgi apparatus produces vesicles that migrate along microtubules and fuse to become a cell plate. d. the inner plasma membrane divides by cytokinesis as in animal cells and then secretes a cellulose cell wall.

c. the Golgi apparatus produces vesicles that migrate along microtubules and fuse to become a cell plate.

As they flow over rotten logs, slime molds appear to lack any partitioning into individual cells; however, slime molds do become cellular when they change form to produce spores. In light of the cell theory that "all living things are composed of cells," then a. these tissues are not living because they are not cellular. b. these tissues are a bridge between nonliving and primitive living cells. c. the general concept still holds because these organisms are cellular at specific stages in their life cycle. d. this proves that a "vital force" beyond cell chemistry can give life to substances.

c. the general concept still holds because these organisms are cellular at specific stages in their life cycle.

All of the following are true regarding transgenic animals EXCEPT a. foreign genes are micro-injected into eggs, fertilized in vitro, and the zygotes are placed in host females to develop. b. the process has created mice that have a phenotype similar to that of a person with cystic fibrosis. c. the process has been used to develop organisms that are partially animal and partially plant. d. the product desired by the transgenic procedure may be secreted in the milk of female offspring.

c. the process has been used to develop organisms that are partially animal and partially plant.

Which type of junctions will create a solid barrier to prevent molecules from moving between the cells? a. gap b. desmosomes c. tight d. Plasmodesmata are narrow channels

c. tight

The organisms on the Galápagos Islands that were most important to the development of Darwin's theory of natural selection were a. rabbits and hares. b. plants and fungi. c. tortoises and finches. d. snails and fish. e. monkeys and armadillos.

c. tortoises and finches.

The following picture depicts which of the following changes in chromosome structure? a. deletion b. duplication c. translocation d. aneuploidy

c. translocation

Which type of covalent bond is the strongest? a. single b. double c. triple d. quadruple e. All covalent bonds are equal in strength.

c. triple

Dr. James isolated Staphylococcus aureus, a type of bacteria, from the leg wound of a ten-year-old boy. He suspected these bacteria would grow better at body temperature than room temperature (72°F), but thought that he should collect data to support his thinking. Dr. James introduced the same number of Staphylococcus bacteria into each of six test tubes containing the same type and amount of nutrient broth. Three test tubes were incubated at 98.6°F (Group 1), while three test tubes (Group 2) sat at 72°F. After 24 hours, Dr. James compared the turbidity (indicative of growth) of all six tubes and rated each on a scale of 0 - 4. 0 indicates no turbidity (no growth), while 4 indicates high turbidity (high growth). The following data were collected: To ensure a controlled experiment, all of the following conditions (variables) should be identical in Group 1 and Group 2: a. type of bacteria, temperature, and incubation period b. temperature and amount of initial inoculum (bacteria used) c. type of bacteria, incubation period, amount of bacteria used d. degree of turbidity, incubation period, and amount of bacteria

c. type of bacteria, incubation period, amount of bacteria used

The cell formed after fertilization of an egg by a sperm is called a(n) a. gamete. b. sperm cell. c. zygote. d. egg cell. e. ovum.

c. zygote.

Which is a true statement about ribosomes? a. Ribosomes contain DNA and protein. b. Ribosomes are active in carbohydrate synthesis. c. Ribosomal subunits leave the nucleus after being formed by the nucleolus. d. Polyribosomes are the subunits of ribosomes. e. Ribosomes are only found associated with the endoplasmic reticulum in prokaryotic cells.

c.Ribosomal subunits leave the nucleus after being formed by the nucleolus.

A solution with a pH of 6 has _____ times _____ OH- than a solution with a pH of 10. a. 40; more b. 4000; less c. 10,000; less d. 4; less e. 10,000 more

d. 10,000; less

If a sperm cell contains 8 chromosomes, it comes from an animal whose regular body cells have _____ chromosomes. a. 4 b. 8 c. 12 d. 16 e. 24

d. 16

If 15% of the bases in a DNA molecule are adenine, what percentage is cytosine? a. 15% b. 30% c. 70% d. 35%

d. 35%

Complete oxidative breakdown of glucose results in _____ ATP molecules. a. 2 b. 4 c. 32 d. 36 e. 39

d. 36

Aerobic cellular respiration yields about ____ of the energy of glucose in ATP molecules. a. 2% b. 15% c. 28% d. 39%

d. 39%

The diploid (2n) number of chromosomes for a human being is a. 23. b. 24. c. 44. d. 46. e. 48.

d. 46.

What is the blending theory of inheritance? a. Mendel's theory of how traits are passed from parent to offspring. b. Darwin's theory of how traits are passed from the parent to the offspring. c.The modern theory of how genetic information is passed from parents to offspring. d. A theory that describes how offspring show traits intermediate between those of the parents.

d. A theory that describes how offspring show traits intermediate between those of the parents.

Meiosis occurs during all of the following EXCEPT a. gametogenesis. b. oogenesis. c. spermatogenesis. d. A, B, and C all involve meiosis. e. Neither A, B, or C. All of the above involve mitosis.

d. A, B, and C all involve meiosis.

Which statement below correctly describes why ATP is a high energy structure? a. ATP contains two phosphate bonds that contain high levels of energy. b. ATP contains hydro-carbon chains that hold energy. c. ATP is composed of thymine which is a high energy molecule. d. ATP has three phosphate bonds, two of which are unstable and easily broken. e. ATP has three phosphate bonds that are very stable and difficult to break.

d. ATP has three phosphate bonds, two of which are unstable and easily broken.

Which of the following is an example of evolution that can be observed and studied? a. bacterial resistance to antibiotics b. resistance of weeds to select herbicides c. resistance of HIV to drug treatment d. All are examples of evolution that can be observed and studied.

d. All are examples of evolution that can be observed and studied.

Which of the following is an example of potential energy? a. a Snickers bar b. an apple c. a glass of milk d. All are examples of potential energy.

d. All are examples of potential energy.

Which of the following observations were made during Darwin's journey aboard the Beagle that led him to believe in evolution? a. geological formations and species variations b. geological formation and the fixation of species c. artificial selection and species variation d. All of the above are correct.

d. All of the above are correct.

Gene therapy in humans has been used to treat a. cystic fibrosis. b. cancer. c. inborn errors of metabolism. d. All of the answer choices. e. Only two of the answer choices.

d. All of the answer choices.

In what field of study would scientists predict a protein's three-dimensional shape and how DNA mutations would affect protein function? a. bioinformatics b. proteomics c. genomics d. All of the answer choices.

d. All of the answer choices.

Which of the following organisms are ultimately dependent on the sun as a source of energy? a. A night-blooming flower that is pollinated by night-flying bats. b. An underground earthworm that avoids the sun. c. A cave fish that feeds on plant debris. d. All of the choices ARE ultimately dependent on the sun. e. All of the choices are NOT ultimately dependent on the sun.

d. All of the choices ARE ultimately dependent on the sun.

Why is it beneficial for pyruvate to be reduced via fermentation when oxygen is not available? a. The organism can survive short spells of anaerobic conditions and maintain growth and reproduction. b. The fermentation reaction regenerates NAD+, which is required for the first step in the energy-harvesting phase of glycolysis. c. Fermentation can provide a rapid burst of ATP. d. All of the choices are advantages. e. None of the choices is an advantage.

d. All of the choices are advantages.

A saturated fat a. is often solid at room temperature. b. has fatty acids with no double bonds between the carbon atoms. c. is of animal origin. d. All of the choices are correct.

d. All of the choices are correct.

Binary fission in bacteria differs from mitosis because a. the chromosome copies attach to the plasma membrane and are pulled apart by cell growth. b. the chromosome is a simple DNA strand without complex proteins and there is no spindle formation. c. there is no nuclear membrane to break down and rebuild. d. All of the choices are correct. e. None of the choices are correct.

d. All of the choices are correct.

Darwin observed that a. members of a population vary in their functional, physical, and behavioral characteristics. b. there is a constant struggle for survival of organisms. c. organisms in a normal population differ in reproductive success. d. All of the choices are correct. e. None of the choices are correct.

d. All of the choices are correct.

Human gene therapy a. includes the insertion of genetic material into human cells for the treatment of a genetic disease. b. has been used for treatment of children who have severe combined immunodeficiency. c. has been used in a trial to treat familial hypercholesterolemia. d. All of the choices are correct.

d. All of the choices are correct.

In a Punnett square, a. all the possible types of sperm are lined up either horizontally or vertically. b. all the possible types of eggs are lined up either horizontally or vertically. c. the results show the offspring's expected genotypes. d. All of the choices are correct. e. Only two of the choices are correct.

d. All of the choices are correct.

On the Galápagos Islands, the amazing tool-using "woodpecker finch" can modify twigs to pry out grubs. Because there are no true woodpeckers on the Galápagos Islands, this behavior allows this finch to exploit an untapped food source. However, not all members of this species exhibit this behavior, which is probably learned from watching other finches. Therefore, a. young chicks isolated at hatching will not know how to do this. b. it is probably not "hardwired" in the brain as a behavior passed on genetically. c. there must be a great advantage to reaching this food source for this learned behavior to be repeated by most descendants of each generation. d. All of the choices are correct.

d. All of the choices are correct.

Single-celled prokaryotes a. lack a membrane-bounded nucleus. b. are classified in the domains Bacteria and Archaea. c. are found in almost all habitats. d. All of the choices are correct.

d. All of the choices are correct.

The component of an animal cell membrane that regulates the fluidity by stiffening the membrane at higher temperatures and preventing the membrane from freezing at lower temperatures is a. cholesterol. b. lipid in nature. c. a steroid. d. All of the choices are correct.

d. All of the choices are correct.

The function of mitosis is a. growth of the organism and tissue repair. b. to ensure that each new cell receives a complete set of genetic information. c. asexual reproduction in some species. d. All of the choices are correct.

d. All of the choices are correct.

Variation within a population is maintained by a. mutation. b. genetic recombination due to fertilization. c. gene flow. d. All of the choices are correct.

d. All of the choices are correct.

Which of the following is required for natural selection to occur in a population? a. heritable variation b. accumulation of adaptive traits c. differential reproduction d. All of the choices are correct.

d. All of the choices are correct.

Saturated fatty acids and unsaturated fatty acids differ in a. the number of carbon-to-carbon bonds. b. the consistency at room temperature. c. the number of hydrogen atoms present. d. All of the choices are differences between saturated and unsaturated fatty acids.

d. All of the choices are differences between saturated and unsaturated fatty acids.

Having similar _____ would allow tissues and organs to be transplanted easily. a. cell recognition proteins b. major histocompatibility complex proteins c. carbohydrate chains in the cell membrane d. All of the choices are involved in tissue transplantation.

d. All of the choices are involved in tissue transplantation.

Which of the following help direct the movement of materials or organelles throughout the cell? a. rough endoplasmic reticulum b. cytoskeleton c. smooth endoplasmic reticulum d. All of the choices are true.

d. All of the choices are true.

Which of the following is/are true about sexual reproduction? a. Gametes are the only haploid phase of the human life cycle. b. The production of gametes is known as gametogenesis. c. A fertilized egg is known as a zygote. d. All of the choices are true. e. None of the choices are true.

d. All of the choices are true.

Living organisms are characterized by a. adapting to the environment. b. evolving over time. c. displaying homeostatic controls. d. All of the choices pertain to living organisms.

d. All of the choices pertain to living organisms.

Which of the following disorders is not a X-linked trait? a. hemophilia b.muscular dystrophy c. adrenoleukodystrophy d. All of these are X-linked disorders.

d. All of these are X-linked disorders.

Which of the following products can be found in vacuoles? a. water b. sugars c. water-soluble pigments d. All of these are found in vacuoles.

d. All of these are found in vacuoles.

Angiogenesis a. is the growth of blood vessels into the tumor. b. is directed by additional mutations in tumor cells. c. brings nutrients and oxygen to a tumor. d. All of these. e. None of these.

d. All of these.

Which of the following are necessary for a cell to pass the G1 checkpoint? a. Growth signals must be present. b. Sufficient nutrients must be available. c. The DNA must be undamaged. d. All of these. e. None of these.

d. All of these.

Which statement describes the currently accepted theory of how an enzyme and its substrate fit together? a. When the product binds to the enzyme, the substrate changes shape to accommodate the reaction. b. The enzyme is like a key that fits into the substrate, which is like a lock. c. The active site is permanently changed by its interaction with the substrate. d. As the substrate binds to the enzyme, the shape of the active site changes to accommodate the reaction.

d. As the substrate binds to the enzyme, the shape of the active site changes to accommodate the reaction.

Which of the following substances attract water molecules? a. a nonpolar substance b. a polar substance c. an ionic substance d. Both polar and ionic substances. e. All of the above will attract water molecules.

d. Both polar and ionic substances.

What do the founder effect and the bottleneck effect have in common? a. Both the founder effect and the bottleneck effect result from mutation. b. Both the founder effect and the bottleneck effect result from an increase in gene flow. c. Both the founder effect and the bottleneck effect are examples of disruptive selection. d. Both the founder effect and the bottleneck effect are forms of genetic drift.

d. Both the founder effect and the bottleneck effect are forms of genetic drift.

Which of the following is the primary function of carbohydrates in living organisms? a. Carbohydrates are used mainly for insulation and protection. b. Carbohydrates store hereditary information. c. Carbohydrates form the basic structural framework of all cell membranes. d. Carbohydrates are used as an immediate energy source.

d. Carbohydrates are used as an immediate energy source.

About 12,000 years ago, a mass extinction event occurred and 75% of the world's large mammal species were eliminated. While a small number of cheetahs managed to survive and restore the world's population of cheetahs, this event caused an extreme reduction of the cheetah's genetic diversity. Based on the principles of natural selection, which of the following is a consequence of their reduced genetic diversity? a. Cheetahs have never fully adapted to the current environment. b. Cheetahs within the same population do not vary in their reproductive success. c. Cheetahs are unable to effectively compete with each other for resources. d. Cheetahs have very little heritable variation, which will make it difficult for them to adapt as the environment changes.

d. Cheetahs have very little heritable variation, which will make it difficult for them to adapt as the environment changes.

Which represents the correct sequence of stages in the cell cycle? a. G1, G2, S, M b. G1, G2, M, S c. G1, M, G2, S d. G1, S, G2, M

d. G1, S, G2, M

Which of these statements is NOT true about DNA? a. It is the genetic material of the cell. b. It forms a double helix. c. Adenine pairs with thymine and guanine pairs with cytosine. d. It contains the sugar ribose. e. The sugar and phosphate groups form the backbone of the molecule.

d. It contains the sugar ribose.

Which is NOT true about the cell theory? a. Its various parts were described by Schleiden, Schwann, and Virchow. b. It states that all organisms are composed of cells. c. It states that all cells come from preexisting cells. d. It states that bacteria and other small organisms can arise spontaneously. e. It is accepted today by biologists as applying to virtually all forms of life.

d. It states that bacteria and other small organisms can arise spontaneously.

Which scientist presented arguments to support a theory of geological change, proposing that the earth was subject to slow but continuous erosion and uplift? a. Leclerc b. Lamarck c. Cuvier d. Lyell

d. Lyell

_____ are to ribosomes as lipids are to _____. a. Carbohydrates; rough endoplasmic reticulum b. Nucleoli; lysosomes c. Sugars; peroxisomes d. Proteins; smooth endoplasmic reticulum

d. Proteins; smooth endoplasmic reticulum

In the case of the peppered moths in England, when Kettlewell set up cameras to document that more white or black moths were eaten by birds on clean or sooty trees, he was verifying which factor involved in evolution by natural selection? a. The organisms vary in traits and that these traits may be acquired during a lifetime. b. The variation is inherited. c. More young are born than can survive, so there will always be competition. d. Some individuals are better adapted to a particular environment than other individuals.

d. Some individuals are better adapted to a particular environment than other individuals.

Which best describes the second law of thermodynamics? a. Energy is not created nor destroyed, but it can change into matter. b. Energy is not created nor destroyed, but it can change from one energy form to another. c. Energy can be created from matter or used to produce matter. d. Some useful energy is lost as heat whenever an energy transfer occurs. e. Energy transfers are always 100% efficient in changing energy from one useful form to another.

d. Some useful energy is lost as heat whenever an energy transfer occurs.

Which statement about the nucleus is incorrect? a. The nucleus contains the genetic material of the cell. b. The nucleus contains the information that ribosomes use to carry out protein synthesis. c. The nucleus contains the instructions for copying itself. d. The nucleus contains the information that ribosomes use to carry out cellular respiration.

d. The nucleus contains the information that ribosomes use to carry out cellular respiration.

Why are plants green? a. They absorb only green wavelengths of light. b. They absorb only yellow and blue wavelengths of light, which when mixed within the leaf, give plants their characteristic green color. c. They reflect all of the ultraviolet wavelengths and absorb all of the visible light spectrum. d. They reflect green wavelengths of light and absorb other colors of the visible light spectrum. e. They reflect the colors of the visible light spectrum and absorb all of the wavelengths associated with the ultraviolet spectrum.

d. They reflect green wavelengths of light and absorb other colors of the visible light spectrum.

What will happen to a celery stick that is placed in a glass of water? a. Salts will move into the celery cells from the surrounding solution. b. Water will move into the celery cells from the surrounding solution. c. Salts will move out of the celery cells into the surrounding solution. d. Water will move out of the celery cells into the surrounding solution. e. None of these choices will occur.

d. Water will move into the celery cells from the surrounding solution.

Which of the following is the reason why plants wilt if they are watered with a salt solution? a. Wilting protects the leaves from full exposure to the salt solution. b. Wilting results from an increase in turgor pressure. c. Wilting results from the salt weakening the plant cell walls. d. Wilting results from the loss of water.

d. Wilting results from the loss of water.

All of the following would denature a protein except a. heating to temperatures above 100° C. b. addition of a strong acid. c. addition of a strong base. d. a missing coenzyme.

d. a missing coenzyme.

Which of the following terms best describes a conceptual scheme in science that is strongly supported, has not yet been found incorrect, and is based on the results of many observations? a. a scientific model b. an experiment c. descriptive research d. a scientific theory or principle e. experimental results

d. a scientific theory or principle

To what does the term chiasma refer? a. the process of fertilization b. structures that hold the chromosomes in alignment on the metaphase plate c. the process of crossing-over d. a structure that holds together homologues during crossing-over e. the period between meiosis I and meiosis II

d. a structure that holds together homologues during crossing-over

A polypeptide has an amino acid sequence of: alanine-leucine-tryptophan-glycine-valine-alanine.This chain of amino acids is further organized into a helix that folds in and around itself to form a globular structure. The primary structure of this polypeptide is a. globular. b. pleated-sheet. c. alpha helix. d. alanine-leucine-tryptophan-glycine-valine-alanine.

d. alanine-leucine-tryptophan-glycine-valine-alanine.

What phase of mitosis is pictured? a. prophase b. prometaphase c. metaphase d. anaphase e. telophase

d. anaphase

Which listing correctly indicates a sequence of increasing biological organization? a. molecule, cell, organelle, atom b. organelle, tissue, cell, molecule c. organ, tissue, atom, molecule d. atom, molecule, organelle, cell

d. atom, molecule, organelle, cell

Fossils like Archaeopteryx offer evidence linking a. reptiles and mammals. b. fish and amphibians. c. birds and mammals. d. birds and dinosaurs. e. amphibians and reptiles.

d. birds and dinosaurs.

The theory of _____ proposed that sudden and widespread disasters that result in mass extinctions followed by repopulation could explain why species change over time. a. artificial selection b. natural selection c. uniformitarianism d. catastrophism

d. catastrophism

What is the smallest unit of living matter? a. atom b. element c. Golgi apparatus d. cell e. nucleus

d. cell

All of the following are true concerning Down syndrome EXCEPT a. it is caused by autosomal trisomy 21. b. in 23% of cases, the sperm contributes the extra chromosome. c. it is the most common trisomy in humans. d. chances of a woman having a child with Down syndrome decrease as she gets older.

d. chances of a woman having a child with Down syndrome decrease as she gets older.

Which of the following organelles is found in autotrophic eukaryotic cells but not heterotrophic eukaryotic cells? a. lysosomes b. ribosomes c. rough endoplasmic reticulum d. chloroplast e. mitochondria

d. chloroplast

The amino acids we cannot synthesize are called _____ because we _____. a. unnecessary; therefore do not need them b. limiting; must include them in our diet c. anabolic; must use alternative amino acids d. essential; must include them in our diet e. superfluous; must survive without them

d. essential; must include them in our diet

Which process produces alcohol or lactate? a. the citric acid cycle b. glycolysis c. the electron transport system d. fermentation e. the preparatory reaction

d. fermentation

Which carbohydrate is used to store energy in the liver? a. starch b. chitin c. cellulose d. glycogen

d. glycogen

Meiosis accomplishes all of the following EXCEPT a. gamete production. b. reduction of chromosome number (from 2n to n). c. providing genetic variation in sexually reproducing organisms. d. growth and repair.

d. growth and repair.

All cells have these two characteristics: a. contain DNA in the nucleus and have a plasma membrane b. produce a cell wall outside of the plasma membrane c. contain mitochondria and chloroplasts d. have a plasma membrane and ribosomes in the cytoplasm

d. have a plasma membrane and ribosomes in the cytoplasm

Which is associated with the inability to produce factor VIII in the blood? a. Williams syndrome b. trisomy 21 c. color blindness d. hemophilia A e. Duchenne muscular dystrophy

d. hemophilia A

A population must have _____ for natural selection to occur. a. a stable environment b. abundant resources c. many individuals d. heritable variation

d. heritable variation

Nucleotides contain all of the following except a. a phosphate group. b. a 5-carbon sugar. c. a nitrogen base. d. histones.

d. histones.

Identify "d." a. inner membrane b. matrix c. cristae d. intermembrane space e. outer membrane

d. intermembrane space

The function of the polymerase chain reaction in genetic engineering is to a. cut DNA into many fragments. b. carry DNA into a new cell. c. link together newly joined fragments of DNA. d. make multiple copies of a specific segment of DNA. e. separate fragments of DNA by their length and electrical charges.

d. make multiple copies of a specific segment of DNA.

Which of the following agents of evolution will help a population become better adapted to its environment? a. genetic drift b. bottleneck c. gene flow d. natural selection e. mutation

d. natural selection

Prior to prescription medications to control stomach acid and heart burn, people consumed baking soda (sodium bicarbonate) to alleviate their symptoms. To be effective, baking soda must: a. effectively buffer stomach acid by releasing H+. b. release water and dilute the stomach acid. c. block acid production by combining with OH-. d. neutralize stomach acid by combining with excess H+.

d. neutralize stomach acid by combining with excess H+.

If the Hardy-Weinberg equilibrium is met, what is the net effect? a. evolution leading to a population better adapted to an unchanging environment b. evolution leading to a population better adapted to a changing environment c. very slow and continuous evolution with no increased adaptation d. no evolution because the alleles in the population remain the same

d. no evolution because the alleles in the population remain the same

Since each child of two heterozygous parents has a 50% chance of receiving a recessive trait from each parent, a. if the first child is phenotypically recessive, then the next child must be phenotypically dominant. b. if the first child is phenotypically recessive, then the next child has a 3/4 chance of being phenotypically recessive. c. if the first child is phenotypically recessive, then the next child has a 1/2 chance of being phenotypically recessive. d. no matter what the first child's phenotype, the next child will have a 1/4 chance of being phenotypically recessive.

d. no matter what the first child's phenotype, the next child will have a 1/4 chance of being phenotypically recessive.

In the scientific method, what precedes the formation of the hypothesis? a. a prediction b. experimentation c. analysis of results d. observation

d. observation

In the reaction NAD --> NADH, the NAD is _____, while the NADH is _____. a. reduced; oxidized b. oxidized; oxidized c. reduced; reduced d. oxidized; reduced

d. oxidized; reduced

Which organelle is primarily responsible for the breakdown of lipids within the cell? a. Golgi apparatus b. chloroplast c. vacuole d. peroxisome

d. peroxisome

Which of the following organisms are capable of photosynthesis? a. plants only b. plants and algae only c. plants and some bacteria only d. plants, algae, and some bacteria

d. plants, algae, and some bacteria

Haiti is settled by peoples of both African and European ancestry. A young couple, both with mixed ancestry, marry and have several children. The children, who all have equal exposure to sunlight, vary widely in the amount of skin melanin. One child is lighter than both parents, and one is darker. The simple explanation for this is a. epistasis. b. multiple alleles are available for the one chromosomal locus that governs skin color. c. the environment influenced the phenotypes of the children. d. polygenic inheritance. e. gene linkage.

d. polygenic inheritance.

While we are sitting down to lunch, we are consuming _____ energy, which will then be converted into _____ energy as we work until dinner time. a. potential; kinetic b. kinetic; potential c. kinetic; free d. potential; stored

d. potential; kinetic

A peptide bond is found in which type of biological molecule? a. carbohydrate b. lipid c. nucleic acid d. protein

d. protein

An atom's atomic mass is best described as the mass of a. the protons it contains. b. the neutrons it contains. c. electrons in the outermost shell. d. protons and neutrons it contains. e. protons and electrons it contains.

d. protons and neutrons it contains.

The process by which cholesterol is transported into the cell by the binding of LDL to its receptor and the internalization of the receptor-LDL complex is a. facilitated transport. b. active transport. c. cotransport. d. receptor-mediated endocytosis. e. exocytosis.

d. receptor-mediated endocytosis.

Human blood has a pH of about 7.4. This is a. neutral. b. very acidic. c. slightly acidic. d. slightly basic.

d. slightly basic.

Where in the human male does spermatogenesis occur? a. ovaries b. prostate gland c. epididymus d. testes e. penis

d. testes

All of the following are examples of natural selection EXCEPT a. the distribution of dark and light colored peppered moths in Britain. b. a rise in bacterial resistance to antibiotics. c. the reduction in beak length of scarlet honeycreepers when they changed food sources. d. the 150 breeds of dogs developed from ancestral wolves. e. two of these are not examples of natural selection.

d. the 150 breeds of dogs developed from ancestral wolves.

The majority of the carbon dioxide we exhale is produced in a. glycolysis. b. the electron transport system. c. lactate fermentation. d. the citric acid cycle.

d. the citric acid cycle.

To what does the term stroma refer? a. the double membrane surrounding the chloroplast b. a flattened disk or sac in the chloroplast c. a stack of thylakoid membrane structures d. the fluid-filled space of the chloroplast e. the pores on the underside of the leaf that facilitate gas exchange

d. the fluid-filled space of the chloroplast

If a pea plant shows a recessive phenotype, a. the genotype may be TT or Tt. b. the genotype may be Tt or tt. c. the genotype can only be TT. d. the genotype can only be tt. e. the genotype may be TT, Tt, or tt.

d. the genotype can only be tt.

Darwin's geometric ratio of increase pertains specifically to a. the inheritable variations found in a population. b. the artificial selection of traits in plants or animals. c. the inheritance of acquired traits. d. the overproduction potential of a species.

d. the overproduction potential of a species.

The activity of an enzyme might be increased by all of the following except a. an increase in substrate concentration. b. a vitamin. c. a two to four degree increase in temperature. d. the presence of other enzymes.

d. the presence of other enzymes.

All of the following are characteristics of oncogenes EXCEPT a. they are mutated proto-oncogenes. b. they cause cancer. c. they are genes that stimulate uncontrolled cell division. d. they are tumor suppressor genes.

d. they are tumor suppressor genes.

Gene pharming is the use of a. transgenic plants to produce larger vegetables and sweeter fruits. b. bacteria for bioremediation. c. STR profiling to determine evolutionary relationships among organisms. d. transgenic farm animals to produce pharmaceuticals.

d. transgenic farm animals to produce pharmaceuticals.

Codons a. are each comprised of a triplet of nucleotides. b. are unambiguous. c. provide the instructions for synthesizing proteins. d. All of the choices are correct.

d.All of the choices are correct.

Which of the following features are present at the ecosystem level? a. chemical cycling through the food chain b. energy flow that begins at the producer level c. input of solar energy d. complex interactions between a variety of populations e. All are features of an ecosystem.

e. All are features of an ecosystem.

Which statement is NOT true about subatomic particles? a. Protons are found in the nucleus. b. Neutrons have no electrical charge. c. Electrons contain much less mass than neutrons. d. Electrons are found in orbitals around the nucleus. e. All electrons in an atom contain the same amount of energy.

e. All electrons in an atom contain the same amount of energy.

An earlier classification system grouped organisms by whether they inhabited the air, land, or sea. More modern classification systems such as the three-domain system are divided into class-order-family-genus-species as described in this chapter. What advantages does the more modern classification system have over the older system? a. The modern classification system better represents the unity of life. b. The modern classification system reflects the evolutionary relationships between organisms. c. The modern classification system allows for the precise organization of a multitude of species. d. The more modern classification system groups organisms based on similarities related to their structure and evolution. e. All of the choices are correct.

e. All of the choices are correct.

Apoptosis a. is programmed cell death. b. is a process that acts to decrease the number of somatic cells. c. frees the fingers and toes of the human embryo from their d. "webbed" structure to independent structures. e. All of the choices are correct.

e. All of the choices are correct.

Plants have been genetically engineered for all of the following reasons EXCEPT a. to have a higher vitamin A content. b. to make the plant more resistant to pests. c. to produce human proteins. d. to reduce food spoilage. e. All of the choices are correct.

e. All of the choices are correct.

Which of the following is/are a biological "population?" a. all of the corn plants in a cornfield b. all of the variable-colored ladybird beetles of the species Harmonia axyridis in a forest c. all male and female English sparrows that reside in your community d. all of the human population of a rural western town e. All of the choices are correct.

e. All of the choices are correct.

Which of the following likely reflects the presence of one or more gene mutations? a. Fruit flies subjected to intense radiation produce a wider array of variable offspring. b. A chemical leaking from the surface of an old abandoned coal mine alters a regulatory gene so that a cricket nymph develops an extra set of eyes. c. The bacteria that cause gonorrhea, a common sexually transmitted disease, have previously been killed by penicillin; however, after continuous usage of the antibiotic, penicillin-resistant strains are now becoming prevalent. d. Radiation causes an alteration in a DNA nucleotide sequence, which is discovered when mapped, but which appears to be neither increasing nor decreasing in successive generations. e. All of the choices are correct.

e. All of the choices are correct.

You are interested in the effect of increased carbon dioxide versus normal air on the growth of corn plants as well as the effect of green light versus full sunlight on the growth of corn plants. Your plan is to set up your experiment inside a greenhouse where you can control the environment. Which of the following is an aspect of the experiment that should be considered and controlled? a. An increase in carbon dioxide should not result in a substantial decrease of other necessary gases. b. All seedlings come from one uniform strain. c. The intensity or brightness of the green light equals the intensity of the full sunlight. d. All temperatures and available water remain the same for all plants. e. All of the choices are important considerations.

e. All of the choices are important considerations.

Which of the following observations is not part of natural selection as posed by Darwin? a. Individuals within a species exhibit variation. b. Organisms compete for available resources. c. Individuals within a population differ in terms of reproductive success. d. Organisms become adapted to conditions as the environment changes. e. All of these observations are part of natural selection as posed by Darwin.

e. All of these observations are part of natural selection as posed by Darwin.

Why did Mendel choose pea plants for his research? a. Peas are easy to cultivate. b. Pea plants have a short generation time. c. Pea plants are self-pollinating but can be cross-fertilized easily. d. Many true-breeding varieties were available. e. All of these were important characteristics in Mendel's selection.

e. All of these were important characteristics in Mendel's selection.

Which stage is most associated with a cell that is unable to divide again, such as a muscle or nerve cell? a. M phase b. G1 phase c. G2 phase d. S phase e. G0 phase

e. G0 phase

If two parents have blood genotypes IAi and IBi, which of the following blood genotypes are possible in their children? a. IAi and IBi only b. IAi, IBi, and IAIB only c. IAIA, IBIB, and IAIB only d. IAi, IBi, and ii only e. IAi, IBi, IAIB and ii

e. IAi, IBi, IAIB and ii

Which of the following is NOT true about daughter cells of mitosis or meiosis? a. In meiosis, daughter cells are haploid. b. In meiosis, there are four daughter cells. c. In mitosis, there are two daughter cells. d. In mitosis, the daughter cells are genetically identical. e. In meiosis, the daughter cells are genetically identical.

e. In meiosis, the daughter cells are genetically identical.

All of the following are associated with translation except a. Growth of a polypeptide chain. b. Attachment of a ribosome to mRNA. c. Binding of two tRNA molecules per ribosome. d. Liberation of polypeptide from the ribosome. e. Production of mRNA.

e. Production of mRNA.

Protons (H+) accumulate in the thylakoid space during electron transport between photosystems II and I. Which of the following describes what will happen to these protons next? a. The protons will reenter the electron transport chain and be used to generate water. b. The protons will be used to oxidize NADPH to NADH, generating ATP in the process. c. The protons will diffuse out of the lipid layer surrounding the thylakoid and exit the plant during gas exchange. d. The protons will raise the pH of the thylakoid space until it becomes alkaline at which point the protons are pumped out of the thylakoid, generating ATP. e. The protons will move from the thylakoid space to the stroma through an ATP synthase complex that generates ATP.

e. The protons will move from the thylakoid space to the stroma through an ATP synthase complex that generates ATP.

The particulate theory of inheritance a. preceded Mendel's research by a century. b. was proposed by Mendel. c. is based on particles or hereditary units we now call genes. d. All of the choices are correct. e. Two of the above choices are correct.

e. Two of the above choices are correct.

At which stage of meiosis is each chromosome composed of a single chromatid? a. prophase I b. metaphase II c. telophase I d. prophase II e. anaphase II

e. anaphase II

If there are twelve different intermediate products produced in the stages of a metabolic pathway within a cell, we can expect that there a. is one enzyme that carries this process through to the end product. b. is one enzyme for degradation and another enzyme for synthesis. c. may not be any enzymes involved if this is a natural cell product. d. must be twelve different raw materials combined in the cell by one enzyme. e. are about twelve enzymes, at least one responsible for each step in the metabolic pathway.

e. are about twelve enzymes, at least one responsible for each step in the metabolic pathway.

Identify the following molecule: a. amino acid b. hydrocarbon c. carbohydrate d. alcohol e. cholesterol

e. cholesterol

The most common lethal genetic disease among Caucasians is a. neurofibromatosis. b. Tay-Sachs disease. c. phenylketonuria. d. albinism. e. cystic fibrosis.

e. cystic fibrosis.

The correct sequence of events in translation is a. initiation, termination, elongation. b. elongation, termination, initiation. c. termination, elongation, initiation. d. elongation, initiation, termination. e. initiation, elongation, termination.

e. initiation, elongation, termination.

The sum of the chemical reactions in a cell constitute a. coupling reactions. b. free energy. c. endergonic reactions only. d. exergonic reactions only. e. metabolism.

e. metabolism.

During which stage of meiosis are the bivalents arranged along the equator of the spindle? a. prophase I b. metaphase II c. anaphase II d. prophase II e. metaphase I

e. metaphase I

Which is the best definition of active transport? a. movement of molecules from an area of their higher concentration to an area of their lower concentration b. movement of water across a semi permeable membrane from an area of higher water concentration to an area of lower water concentration c. movement of molecules from an area of their lower concentration to an area of their higher concentration d. movement of water across a semi permeable membrane from an area of lower water concentration to an area of higher water concentration e. movement of a substance against its concentration through the release of energy from ATP

e. movement of a substance against its concentration through the release of energy from ATP

Identify "c." a. inner membrane b. matrix c. cristae d. intermembrane space e. outer membrane

e. outer membrane

Membrane-bounded vesicles that contain enzymes that break down fatty acids while producing hydrogen peroxide as a waste product are called a. vacuoles. b. vesicles. c. glyoxysomes. d. lysosomes. e. peroxisomes.

e. peroxisomes.

In a phospholipid bilayer, the a. phosphate groups are hydrophobic. b. fatty acid tails are ionized. c. fatty acid tails are hydrophilic. d. proteins are located only between the two layers. e. phosphate heads are oriented toward the exterior of the cell or toward the cytoplasm.

e. phosphate heads are oriented toward the exterior of the cell or toward the cytoplasm.

Which of the following molecules forms lengths of DNA with "sticky ends"? a. DNA ligase b. DNA polymerase c. RNA polymerase d. reverse transcriptase e. restriction enzyme

e. restriction enzyme

What is the function of gel electrophoresis in genetic engineering? a. cut DNA into many fragments b. carry DNA into a new cell c. link together newly joined fragments of DNA d. make millions of copies of a specific segment of DNA e. separate fragments of DNA by their length and electrical charges

e. separate fragments of DNA by their length and electrical charges

Which of the following is NOT a function of proteins? a. support b. transportation c. defense d. motion e. storage of energy

e. storage of energy

If the small groups of Viking explorers in Greenland and North America had survived and given rise to new populations, such a scenario would be an example of a. gene flow from continent to continent. b. the bottleneck effect. c. genetic drift among the original Viking explorers. d. directional selection. e. the founder effect.

e. the founder effect.

The distribution of species can help provide information about all of the following except a. past geological events. b. the movement of continents. c. formation of volcanic islands. d. ecological change. e. the time period in which a species was present on Earth.

e. the time period in which a species was present on Earth.

A researcher is studying the effects of a new drug on the endomembrane system. She notices the Golgi apparatus is not receiving any proteins and that it is not able to ship any proteins. What part of the endomembrane system was probably affected by this new drug? a. mitochondria b. smooth endoplasmic reticulum c. ribosomes d. nucleus e. vesicles

e. vesicles

In human females, when is meiosis II completed? a. at ovulation b. immediately after the sperm penetrates the secondary oocyte c. immediately after the sperm penetrates the primary oocyte d. after the zygote has formed

immediately after the sperm penetrates the secondary oocyte


Ensembles d'études connexes

Chapter 14 responding to personal and work-related stress

View Set